CVX error: "Disciplined convex programming error: Invalid quadratic form(s): not a square."

Error using .* (line 262)
Disciplined convex programming error:
Invalid quadratic form(s): not a square.

Error in * (line 36)
z = feval( oper, x, y );

Error in final_project2 (line 92)
((-x_n(1)^2+2x_n(1)x(1))(2/s_n(1)-s(1)/(s_n(1)^2)))-(2real((w’A_13w_n)/(q_n(3)))-((w_n’A_13w_n)/(q_n(3)^2)*q(3)))<=0;

how can i fix this error?
please help me,

You haven’t told us which variables are CVX variables as opposed to MATLAB variables, nor the values of the MATLAB variables. Have you proven that the left-hand-side is convex?

here is the CVX part of my code:

cvx_begin
variables q(N) w(NN) s(N) x(N) u(N) t0(1)
maximize(sum(log(u)/log(2)))
subject to
t0>0;
for i=1:N
u(i)-1-2./s_n+s./(s_n.^2)<=0;
u(i)>0;
end
((-x_n(1)^2+2
x_n(1)x(1))(2/s_n(1)-s(1)/(s_n(1)^2)))-(2real((w’A_13w_n)/(q_n(3)))-((w_n’A_13w_n)/(q_n(3)^2)q(3)))<=0;
((-x_n(2)^2+2
x_n(2)x(2))(2/s_n(2)-s(2)/(s_n(2)^2)))-(2
real((w’A_24w_n)/(q_n(4)))-((w_n’A_24w_n)/(q_n(4)^2)q(4)))<=0;
((-x_n(3)^2+2
x_n(3)x(3))(2/s_n(3)-s(3)/(s_n(3)^2)))-(2real((w’A_31w_n)/(q_n(1)))-((w_n’A_31w_n)/(q_n(1)^2)q(1)))<=0;
((-x_n(4)^2+2
x_n(4)x(4))(2/s_n(4)-s(4)/(s_n(4)^2)))-(2
real((w’A_42w_n)/(q_n(2)))-((w_n’A_42w_n)/(q_n(2)^2)q(2)))<=0;
((w’A_12w)/(q(2)))+((w’A_14w)/(q(4)))+sigma^2
(w’B_1w)+sigma^2-(-x_n(1)^2+2x_n(1)x(1))<=0;
((w’A_21w)/(q(1)))+((w’A_23w)/(q(3)))+sigma^2
(w’B_2w)+sigma^2-(-x_n(2)^2+2
x_n(2)x(2))<=0;
((w’A_32w)/(q(2)))+((w’A_34w)/(q(4)))+sigma^2
(w’B_3w)+sigma^2-(-x_n(3)^2+2x_n(3)x(3))<=0;
((w’A_41w)/(q(1)))+((w’A_43w)/(q(3)))+sigma^2
(w’B_4w)+sigma^2-(-x_n(4)^2+2
x_n(4)x(4))<=0;
es
sum(1/q)+er*((w’D_1w)/(q(1))+(w’D_2w)/(q(2))+(w’D_3w)/(q(3))+(w’D_4w)/(q(4))+sigma^2*w’w)+Pcir-(2/t0_n-t0/t0_n^2)<=0;
for i=1:N
s(i)>0;
q(i)>=1/P(e);
end
(abs(g_11)^2/q(1)+abs(g_21)^2/q(2)+abs(g_31)^2/q(3)+abs(g_41)^2/q(4))<=I;
((w’D_1w)/q(1)+(w’D_2w)/q(2)+(w’D_3w)/q(3)+(w’D_4w)/q(4)+sigma^2
w’w)<=P(e);
((w’C_11w)/q(1)+(w’C_21w)/q(2)+(w’C_31w)/q(3)+(w’C_41w)/q(4)+sigma^2
w’E_1w)<=I;
cvx_end

becauce the optimisation problem is from a paper, so i think it’s convex.